Computing a series sum [duplicate]How to prove $sum_n=0^infty fracn^22^n = 6$?Writing an infinite series as the sum of the seriesComputing the sum of an infinite seriescomputing the series $sum_n=1^infty frac1n^2 2^n$Sum of a series to an exact answerConfused computing sum of Fourier seriesClosed form of this series?Prove that the given series is convergent.Sum of reciprocals of the square roots of the first N Natural NumbersSum function for power seriesShowing the sum of a power series is less than P$x$

Surface of the 3x3x3 cube as a graph

Anatomically correct Guivre

How do I write real-world stories separate from my country of origin?

why "American-born", not "America-born"?

Writing "hahaha" versus describing the laugh

"Official wife" or "Formal wife"?

Does attacking (or having a rider attack) cancel Charge/Pounce-like abilities?

Efficient Algorithms for Destroyed Document Reconstruction

Why "strap-on" boosters, and how do other people say it?

Is it normal to "extract a paper" from a master thesis?

Split into three!

Is there a solution to paying high fees when opening and closing lightning channels once we hit a fee only market?

Download app bundles from App Store to run on iOS Emulator on Mac

Illustrating that universal optimality is stronger than sphere packing

What happens when redirecting with 3>&1 1>/dev/null?

What is the required burn to keep a satellite at a Lagrangian point?

Shell builtin `printf` line limit?

VHDL: Why is it hard to desgin a floating point unit in hardware?

Are there historical examples of audiences drawn to a work that was "so bad it's good"?

Negative impact of having the launch pad away from the Equator

What pc resources are used when bruteforcing?

Managing heat dissipation in a magic wand

How does the Earth's center produce heat?

Is the default 512 byte physical sector size appropriate for SSD disks under Linux?



Computing a series sum [duplicate]


How to prove $sum_n=0^infty fracn^22^n = 6$?Writing an infinite series as the sum of the seriesComputing the sum of an infinite seriescomputing the series $sum_n=1^infty frac1n^2 2^n$Sum of a series to an exact answerConfused computing sum of Fourier seriesClosed form of this series?Prove that the given series is convergent.Sum of reciprocals of the square roots of the first N Natural NumbersSum function for power seriesShowing the sum of a power series is less than P$x$













3












$begingroup$



This question already has an answer here:



  • How to prove $sum_n=0^infty fracn^22^n = 6$?

    6 answers



$$sum_k=0^infty frack^23^k$$



I tried with that 2 method but I couldn't get the $n^2$ term.










share|cite|improve this question









New contributor



Erinç Emre Çelikten is a new contributor to this site. Take care in asking for clarification, commenting, and answering.
Check out our Code of Conduct.






$endgroup$



marked as duplicate by Hans Lundmark, Yanior Weg, Lord Shark the Unknown, Lee David Chung Lin, Cesareo May 16 at 6:59


This question has been asked before and already has an answer. If those answers do not fully address your question, please ask a new question.













  • 3




    $begingroup$
    You'll need to take two derivatives here and also make use of the first derivative case since taking two derivatives gives you $k(k-1) = k^2 - k$.
    $endgroup$
    – Cameron Williams
    May 14 at 23:24











  • $begingroup$
    math.stackexchange.com/questions/593996/…
    $endgroup$
    – lab bhattacharjee
    May 15 at 2:07










  • $begingroup$
    Welcome to Mathematics Stack Exchange! A quick tour will enhance your experience. Here are helpful tips to write a good question and write a good answer. For equations, use MathJax.
    $endgroup$
    – dantopa
    May 16 at 1:52















3












$begingroup$



This question already has an answer here:



  • How to prove $sum_n=0^infty fracn^22^n = 6$?

    6 answers



$$sum_k=0^infty frack^23^k$$



I tried with that 2 method but I couldn't get the $n^2$ term.










share|cite|improve this question









New contributor



Erinç Emre Çelikten is a new contributor to this site. Take care in asking for clarification, commenting, and answering.
Check out our Code of Conduct.






$endgroup$



marked as duplicate by Hans Lundmark, Yanior Weg, Lord Shark the Unknown, Lee David Chung Lin, Cesareo May 16 at 6:59


This question has been asked before and already has an answer. If those answers do not fully address your question, please ask a new question.













  • 3




    $begingroup$
    You'll need to take two derivatives here and also make use of the first derivative case since taking two derivatives gives you $k(k-1) = k^2 - k$.
    $endgroup$
    – Cameron Williams
    May 14 at 23:24











  • $begingroup$
    math.stackexchange.com/questions/593996/…
    $endgroup$
    – lab bhattacharjee
    May 15 at 2:07










  • $begingroup$
    Welcome to Mathematics Stack Exchange! A quick tour will enhance your experience. Here are helpful tips to write a good question and write a good answer. For equations, use MathJax.
    $endgroup$
    – dantopa
    May 16 at 1:52













3












3








3





$begingroup$



This question already has an answer here:



  • How to prove $sum_n=0^infty fracn^22^n = 6$?

    6 answers



$$sum_k=0^infty frack^23^k$$



I tried with that 2 method but I couldn't get the $n^2$ term.










share|cite|improve this question









New contributor



Erinç Emre Çelikten is a new contributor to this site. Take care in asking for clarification, commenting, and answering.
Check out our Code of Conduct.






$endgroup$





This question already has an answer here:



  • How to prove $sum_n=0^infty fracn^22^n = 6$?

    6 answers



$$sum_k=0^infty frack^23^k$$



I tried with that 2 method but I couldn't get the $n^2$ term.





This question already has an answer here:



  • How to prove $sum_n=0^infty fracn^22^n = 6$?

    6 answers







sequences-and-series power-series






share|cite|improve this question









New contributor



Erinç Emre Çelikten is a new contributor to this site. Take care in asking for clarification, commenting, and answering.
Check out our Code of Conduct.










share|cite|improve this question









New contributor



Erinç Emre Çelikten is a new contributor to this site. Take care in asking for clarification, commenting, and answering.
Check out our Code of Conduct.








share|cite|improve this question




share|cite|improve this question








edited May 14 at 23:24









David G. Stork

12.7k41837




12.7k41837






New contributor



Erinç Emre Çelikten is a new contributor to this site. Take care in asking for clarification, commenting, and answering.
Check out our Code of Conduct.








asked May 14 at 23:21









Erinç Emre ÇeliktenErinç Emre Çelikten

183




183




New contributor



Erinç Emre Çelikten is a new contributor to this site. Take care in asking for clarification, commenting, and answering.
Check out our Code of Conduct.




New contributor




Erinç Emre Çelikten is a new contributor to this site. Take care in asking for clarification, commenting, and answering.
Check out our Code of Conduct.






marked as duplicate by Hans Lundmark, Yanior Weg, Lord Shark the Unknown, Lee David Chung Lin, Cesareo May 16 at 6:59


This question has been asked before and already has an answer. If those answers do not fully address your question, please ask a new question.









marked as duplicate by Hans Lundmark, Yanior Weg, Lord Shark the Unknown, Lee David Chung Lin, Cesareo May 16 at 6:59


This question has been asked before and already has an answer. If those answers do not fully address your question, please ask a new question.









  • 3




    $begingroup$
    You'll need to take two derivatives here and also make use of the first derivative case since taking two derivatives gives you $k(k-1) = k^2 - k$.
    $endgroup$
    – Cameron Williams
    May 14 at 23:24











  • $begingroup$
    math.stackexchange.com/questions/593996/…
    $endgroup$
    – lab bhattacharjee
    May 15 at 2:07










  • $begingroup$
    Welcome to Mathematics Stack Exchange! A quick tour will enhance your experience. Here are helpful tips to write a good question and write a good answer. For equations, use MathJax.
    $endgroup$
    – dantopa
    May 16 at 1:52












  • 3




    $begingroup$
    You'll need to take two derivatives here and also make use of the first derivative case since taking two derivatives gives you $k(k-1) = k^2 - k$.
    $endgroup$
    – Cameron Williams
    May 14 at 23:24











  • $begingroup$
    math.stackexchange.com/questions/593996/…
    $endgroup$
    – lab bhattacharjee
    May 15 at 2:07










  • $begingroup$
    Welcome to Mathematics Stack Exchange! A quick tour will enhance your experience. Here are helpful tips to write a good question and write a good answer. For equations, use MathJax.
    $endgroup$
    – dantopa
    May 16 at 1:52







3




3




$begingroup$
You'll need to take two derivatives here and also make use of the first derivative case since taking two derivatives gives you $k(k-1) = k^2 - k$.
$endgroup$
– Cameron Williams
May 14 at 23:24





$begingroup$
You'll need to take two derivatives here and also make use of the first derivative case since taking two derivatives gives you $k(k-1) = k^2 - k$.
$endgroup$
– Cameron Williams
May 14 at 23:24













$begingroup$
math.stackexchange.com/questions/593996/…
$endgroup$
– lab bhattacharjee
May 15 at 2:07




$begingroup$
math.stackexchange.com/questions/593996/…
$endgroup$
– lab bhattacharjee
May 15 at 2:07












$begingroup$
Welcome to Mathematics Stack Exchange! A quick tour will enhance your experience. Here are helpful tips to write a good question and write a good answer. For equations, use MathJax.
$endgroup$
– dantopa
May 16 at 1:52




$begingroup$
Welcome to Mathematics Stack Exchange! A quick tour will enhance your experience. Here are helpful tips to write a good question and write a good answer. For equations, use MathJax.
$endgroup$
– dantopa
May 16 at 1:52










3 Answers
3






active

oldest

votes


















5












$begingroup$

$$frac11-x=sum_k=0^inftyx^k$$
$$frac1(1-x)^2=sum_k=1^inftykx^k-1$$
Multiply by $x$
$$fracx(1-x)^2=sum_k=1^inftykx^k$$
$$left(fracx(1-x)^2right)'=sum_k=1^inftyk^2x^k-1$$
Multiply by $x$
$$xleft(fracx(1-x)^2right)'=sum_k=1^inftyk^2x^k$$
then let $x=frac13$






share|cite|improve this answer











$endgroup$




















    0












    $begingroup$

    Hint try to show
    begineqnarray*
    sum_k=0^infty k^2 x^k =fracx(1+4x+x^2)(1-x)^3.
    endeqnarray*






    share|cite|improve this answer









    $endgroup$




















      0












      $begingroup$

      There is another way to deal with this problem.



      Denote $S_n=sum_k=0^n frack^23^k$ then$frac13S_n=sum_k=1^n+1 frac(k-1)^23^k$ so $frac23S_n=sum_k=1^nfrac2k-13^k-fracn^23^n+1$.



      In this way we change the numerator from twice power of $k$ to the lower power. Using the same operation you can change the numerator to numbers without the appearance of $k$ then you can use the summation formula for geometric series.






      share|cite|improve this answer









      $endgroup$



















        3 Answers
        3






        active

        oldest

        votes








        3 Answers
        3






        active

        oldest

        votes









        active

        oldest

        votes






        active

        oldest

        votes









        5












        $begingroup$

        $$frac11-x=sum_k=0^inftyx^k$$
        $$frac1(1-x)^2=sum_k=1^inftykx^k-1$$
        Multiply by $x$
        $$fracx(1-x)^2=sum_k=1^inftykx^k$$
        $$left(fracx(1-x)^2right)'=sum_k=1^inftyk^2x^k-1$$
        Multiply by $x$
        $$xleft(fracx(1-x)^2right)'=sum_k=1^inftyk^2x^k$$
        then let $x=frac13$






        share|cite|improve this answer











        $endgroup$

















          5












          $begingroup$

          $$frac11-x=sum_k=0^inftyx^k$$
          $$frac1(1-x)^2=sum_k=1^inftykx^k-1$$
          Multiply by $x$
          $$fracx(1-x)^2=sum_k=1^inftykx^k$$
          $$left(fracx(1-x)^2right)'=sum_k=1^inftyk^2x^k-1$$
          Multiply by $x$
          $$xleft(fracx(1-x)^2right)'=sum_k=1^inftyk^2x^k$$
          then let $x=frac13$






          share|cite|improve this answer











          $endgroup$















            5












            5








            5





            $begingroup$

            $$frac11-x=sum_k=0^inftyx^k$$
            $$frac1(1-x)^2=sum_k=1^inftykx^k-1$$
            Multiply by $x$
            $$fracx(1-x)^2=sum_k=1^inftykx^k$$
            $$left(fracx(1-x)^2right)'=sum_k=1^inftyk^2x^k-1$$
            Multiply by $x$
            $$xleft(fracx(1-x)^2right)'=sum_k=1^inftyk^2x^k$$
            then let $x=frac13$






            share|cite|improve this answer











            $endgroup$



            $$frac11-x=sum_k=0^inftyx^k$$
            $$frac1(1-x)^2=sum_k=1^inftykx^k-1$$
            Multiply by $x$
            $$fracx(1-x)^2=sum_k=1^inftykx^k$$
            $$left(fracx(1-x)^2right)'=sum_k=1^inftyk^2x^k-1$$
            Multiply by $x$
            $$xleft(fracx(1-x)^2right)'=sum_k=1^inftyk^2x^k$$
            then let $x=frac13$







            share|cite|improve this answer














            share|cite|improve this answer



            share|cite|improve this answer








            edited May 14 at 23:33









            clathratus

            5,7051443




            5,7051443










            answered May 14 at 23:29









            E.H.EE.H.E

            17.9k11969




            17.9k11969





















                0












                $begingroup$

                Hint try to show
                begineqnarray*
                sum_k=0^infty k^2 x^k =fracx(1+4x+x^2)(1-x)^3.
                endeqnarray*






                share|cite|improve this answer









                $endgroup$

















                  0












                  $begingroup$

                  Hint try to show
                  begineqnarray*
                  sum_k=0^infty k^2 x^k =fracx(1+4x+x^2)(1-x)^3.
                  endeqnarray*






                  share|cite|improve this answer









                  $endgroup$















                    0












                    0








                    0





                    $begingroup$

                    Hint try to show
                    begineqnarray*
                    sum_k=0^infty k^2 x^k =fracx(1+4x+x^2)(1-x)^3.
                    endeqnarray*






                    share|cite|improve this answer









                    $endgroup$



                    Hint try to show
                    begineqnarray*
                    sum_k=0^infty k^2 x^k =fracx(1+4x+x^2)(1-x)^3.
                    endeqnarray*







                    share|cite|improve this answer












                    share|cite|improve this answer



                    share|cite|improve this answer










                    answered May 14 at 23:30









                    Donald SplutterwitDonald Splutterwit

                    23.5k21448




                    23.5k21448





















                        0












                        $begingroup$

                        There is another way to deal with this problem.



                        Denote $S_n=sum_k=0^n frack^23^k$ then$frac13S_n=sum_k=1^n+1 frac(k-1)^23^k$ so $frac23S_n=sum_k=1^nfrac2k-13^k-fracn^23^n+1$.



                        In this way we change the numerator from twice power of $k$ to the lower power. Using the same operation you can change the numerator to numbers without the appearance of $k$ then you can use the summation formula for geometric series.






                        share|cite|improve this answer









                        $endgroup$

















                          0












                          $begingroup$

                          There is another way to deal with this problem.



                          Denote $S_n=sum_k=0^n frack^23^k$ then$frac13S_n=sum_k=1^n+1 frac(k-1)^23^k$ so $frac23S_n=sum_k=1^nfrac2k-13^k-fracn^23^n+1$.



                          In this way we change the numerator from twice power of $k$ to the lower power. Using the same operation you can change the numerator to numbers without the appearance of $k$ then you can use the summation formula for geometric series.






                          share|cite|improve this answer









                          $endgroup$















                            0












                            0








                            0





                            $begingroup$

                            There is another way to deal with this problem.



                            Denote $S_n=sum_k=0^n frack^23^k$ then$frac13S_n=sum_k=1^n+1 frac(k-1)^23^k$ so $frac23S_n=sum_k=1^nfrac2k-13^k-fracn^23^n+1$.



                            In this way we change the numerator from twice power of $k$ to the lower power. Using the same operation you can change the numerator to numbers without the appearance of $k$ then you can use the summation formula for geometric series.






                            share|cite|improve this answer









                            $endgroup$



                            There is another way to deal with this problem.



                            Denote $S_n=sum_k=0^n frack^23^k$ then$frac13S_n=sum_k=1^n+1 frac(k-1)^23^k$ so $frac23S_n=sum_k=1^nfrac2k-13^k-fracn^23^n+1$.



                            In this way we change the numerator from twice power of $k$ to the lower power. Using the same operation you can change the numerator to numbers without the appearance of $k$ then you can use the summation formula for geometric series.







                            share|cite|improve this answer












                            share|cite|improve this answer



                            share|cite|improve this answer










                            answered May 14 at 23:37









                            Feng ShaoFeng Shao

                            19911




                            19911













                                Popular posts from this blog

                                Get product attribute by attribute group code in magento 2get product attribute by product attribute group in magento 2Magento 2 Log Bundle Product Data in List Page?How to get all product attribute of a attribute group of Default attribute set?Magento 2.1 Create a filter in the product grid by new attributeMagento 2 : Get Product Attribute values By GroupMagento 2 How to get all existing values for one attributeMagento 2 get custom attribute of a single product inside a pluginMagento 2.3 How to get all the Multi Source Inventory (MSI) locations collection in custom module?Magento2: how to develop rest API to get new productsGet product attribute by attribute group code ( [attribute_group_code] ) in magento 2

                                Category:9 (number) SubcategoriesMedia in category "9 (number)"Navigation menuUpload mediaGND ID: 4485639-8Library of Congress authority ID: sh85091979ReasonatorScholiaStatistics

                                Magento 2.3: How do i solve this, Not registered handle, on custom form?How can i rewrite TierPrice Block in Magento2magento 2 captcha not rendering if I override layout xmlmain.CRITICAL: Plugin class doesn't existMagento 2 : Problem while adding custom button order view page?Magento 2.2.5: Overriding Admin Controller sales/orderMagento 2.2.5: Add, Update and Delete existing products Custom OptionsMagento 2.3 : File Upload issue in UI Component FormMagento2 Not registered handleHow to configured Form Builder Js in my custom magento 2.3.0 module?Magento 2.3. How to create image upload field in an admin form